LSAT and Law School Admissions Forum

Get expert LSAT preparation and law school admissions advice from PowerScore Test Preparation.

User avatar
 Dave Killoran
PowerScore Staff
  • PowerScore Staff
  • Posts: 5853
  • Joined: Mar 25, 2011
|
#41160
Complete Question Explanation
(The complete setup for this game can be found here: lsat/viewtopic.php?t=11758)

The correct answer choice is (C)

The correct answer is a student who can be placed in any of the three classes. As the level 1 class is very restricted, begin your analysis there. I and G are fixed in level 1, and so neither is a consideration. But, the one remaining spot must be occupied by J or K, and so either J or K must be the correct answer (because none of the other students can be in the level 1 class).

Based on the setup discussion and as confirmed in question #2, J cannot be in the level 3 class, and thus answer choice (B) is eliminated. At this point, we know that K must be the correct answer, and therefore (C) is correct.

If the above did not make perfect sense, consider the following: F and L both appear near the end of the chain, and each must score lower than at least G, I, and K. Since G, I, and K are always ahead of F and L, F and L can never appear in the level 1 class. It follows that answer choices (A) and (D) are eliminated. M can never appear in the level 1 class since G, I, and J must always appear ahead of him and therefore answer choice (E) is incorrect.

Get the most out of your LSAT Prep Plus subscription.

Analyze and track your performance with our Testing and Analytics Package.